LSAT and Law School Admissions Forum

Get expert LSAT preparation and law school admissions advice from PowerScore Test Preparation.

Lesson help relating to our Advanced Logical Reasoning Course.
 T.B.Justin
  • Posts: 194
  • Joined: Jun 01, 2018
|
#62423
I want to add to the reasoning of the correct answer choice (C) and I would love a check on my reasoning:

"Will sales of the luxury items subject to the proposed tax occur at currents rates once the proposed tax on luxury items has been passed."

If, the answer is No, and the sales decline, I think that would likely affect non-wealthy individuals, such as, the expert trades craftsman that are contributing to the assembly of the yachts, that is used as a scenario in explaining this answer choice. And since the conclusion is also concerned with affects of only the wealthy individuals and corporations that can afford to purchase such items, if something has an affect on the non-wealthy or non-corporations, that produces an effect on this conclusion, thus passing the variance test?

Thanks :)
 Jay Donnell
PowerScore Staff
  • PowerScore Staff
  • Posts: 144
  • Joined: Jan 09, 2019
|
#63112
Hey!

Yes, you are totally correct here. If the Richie Riches of the world no longer purchase these luxury items due to the tax, then that would absolutely impact the non-wealthy segments of society. You're also bang on to further highlight the tailors, contractors and other craftsmen who would specifically be disadvantaged if the mega-rich stopped buying yachts and fur coats and other frivolous goods.

If the rich continue purchasing goods, then all's well in middle-class-ville, but if they don't, then this would have a negative effect on people outside the 1%. This means that C does exactly what we want it to for an answer choice going through The Variance Test, so it makes C a correct (and verifiable!) response.

Strong work!

Get the most out of your LSAT Prep Plus subscription.

Analyze and track your performance with our Testing and Analytics Package.